A food bank sends a truck with 400 pounds of food to each of five local food pantries every week, and receives a government surplus shipment of 3,200 pounds every other week. If it has 10,000 pounds of food on hand at the begining of July , how much will it have six weeks later?

Answers

Answer 1

Answer:

6,400

Step-by-step explanation:

The calculation is shown below:-

But before that find the total number of pound in five local bonds

= 400 × 5

= 2,000

Now the amount at six week later is

= Food pounds + 3 × number of pounds + 3 × suplus shipment

= 10,000 + 3 × 2,000 - 3 × 3,200

= 10,000 + 6,000 - 9,600

= 6,400

Therefore according to the given situation the correct answer is 6,400


Related Questions

7
Find the product of _m²n,
32
gm²n, -m
-mn³, 14mn².
49

7Find the product of _mn,32gmn, -m-mn, 14mn.49

Answers

Answer: A

Step-by-step explanation:

\(\left(\frac{7}{8} \right)\left(-\frac{32}{49} \right)(14)(m^{2}n)(mn^{3})(mn^{2})=\boxed{8m^{4}n^{6}}\)

If (m-1÷m)=3 , find the value of m^3 - 1÷m^3​

Answers

The value of given equation i.e. \(m^{3} -\frac{1}{m^{3} }\) is \(\frac{63}{8}\)

Given a equation i.e.\(\frac{m-1}{m}\)=3 and asked  to calculate the value of the equation  \(m^{3} -\frac{1}{m^{3} }\)

To calculate the value of   \(m^{3} -\frac{1}{m^{3} }\) , Need to use various operations like subtraction and division)

Given .\(\frac{m-1}{m}\)=3

⇒m-1=3m ( cross multiplies 3 and m)

⇒2m=-1( subtraction is used)

⇒m=-\(\frac{1}{2}\)( division is used)

Thefore the value of m in the given equation i.e.\(\frac{m-1}{m}\)=3 is -\(\frac{1}{2}\)

⇒ \(m^{3} -\frac{1}{m^{3} }\)

Here the value of m =-\(\frac{1}{2}\)

Substituting the value of m in the given equation

⇒\((\frac{-1}{2} )^{3}\)-\((-2)^{3}\)

⇒\(\frac{-1}{8}\)+8(addition is used)

⇒\(\frac{64-1}{8}\)

⇒\(\frac{63}{8}\)

Therefore, the value of the given equation i.e.  \(m^{3} -\frac{1}{m^{3} }\) is \(\frac{63}{8}\)

Learn more about equation here:

brainly.com/question/10413253

#SPJ9

d²v dt² v=2t² +7t+11 Find

Answers

The second derivative of v with respect to t, denoted as d²v/dt², is equal to 4

The second derivative of v with respect to t, we will differentiate v twice.

v = 2t² + 7t + 11

First, let's find the first derivative of v with respect to t (dv/dt):

dv/dt = d/dt (2t² + 7t + 11)

Using the power rule of differentiation, we differentiate each term separately:

dv/dt = 2(2t) + 7(1) + 0

dv/dt = 4t + 7

Now, let's find the second derivative of v with respect to t (d²v/dt²):

d²v/dt² = d/dt (4t + 7)

Again, using the power rule of differentiation, we differentiate each term separately:

d²v/dt² = 4(1) + 0

d²v/dt² = 4

Therefore, the second derivative of v with respect to t, denoted as d²v/dt², is equal to 4.

To know more about second derivative click here :

https://brainly.com/question/29090070

#SPJ4

One year, an estimated 6,955 sandhill cranes migrated in March. The next March, an estimated 3,480 sandhill cranes migrated. To the nearest percent, what is the percent change in the number of migrating cranes from the first March to the next? Decreases should be negative numbers and increases should be positive numbers. Do not include the percent in your answer

Answers

Rounding to the nearest percent, we get that the percent change in the number of migrating cranes from the first March to the next is approximately -50%.

How is a percentage calculated?

To calculate the percentage, we must first divide the amount by the total value and then multiply the result by 100.

To find the percent change in the number of migrating cranes from the first March to the next, we can use the formula:

percent change = ((new value - old value) / old value) × 100%

where the old value is the number of migrating cranes in the first March and the new value is the number of migrating cranes in the next March.

Substituting the given values, we get:

percent change = ((3480 - 6955) / 6955) × 100%

Simplifying, we get:

\percent change = (-3475 / 6955) × 100%

percent change ≈ -50%

Rounding to the nearest percent, we get that the percent change in the number of migrating cranes from the first March to the next is approximately -50%.

To know more about percentage visit:

https://brainly.com/question/29306119

#SPJ1

(10.5 - 7) + (4x3) evaluate the expression

Answers

(10.5 - 7) + ( 4 x 3 ) = 3.5 + 12 = 15.5
Ok done. Thank to me :>

The average weight of a group of seven boys is 56 kg. The individual weights (in kg) of 6 of them are 52,57,55,60,59 and 55. find the weight of the seventh boy​

Answers

Answer:

54 kg

Step-by-step explanation:

Total weight of 7 boys is 392 kg. And other 6 boys total weight is 338 kg.

So the 7th boy's weight is (392-338)=54 kg

Answer:

54

Step-by-step explanation:

338÷6=56.33

so it means that 7th boy is less than 56

338+54=392

392÷7=56

A piano teacher charges students $20.25 each month, plus $5.25 per hour for lessons.
Which expression and value represent the total the instructor will charge a student for hours of lessons in a month
and the amount the instructor would charge a student for 15 hours of lessons in a month?
$20.25 -2; $15
$20.25 +x: $25.50
O $20.25 - $5.25x; $58.50
O $20.25 + $5.25x; $99

Answers

Answer:

The answer should be D. $20.25+$5.25x; $99

Step-by-step explanation:

y=mx+b

m=$5.25

x=15

b=$20.25

This gives us:

5.25x + 20.25

5.25(15)+20.25

78.75+20.25

y=99

Answer: 20.25 + 5.25x ; 99

Hope this helps!

Draw the image located at (-1,6), (2,4), and (1,2). Then for the following mapping. (x,y)>(x-5,y-3). type of mapping:

Draw the image located at (-1,6), (2,4), and (1,2). Then for the following mapping. (x,y)>(x-5,y-3).

Answers

SOLUTION

Given the question in the image, the following are the solution steps to answer the question.

STEP 1: Write the given points

\((-1,6),(2,4),(1,2)\)

STEP 2: Plot these points

STEP 3: Get the new points from the given function

\(\begin{gathered} (x,y)\rightarrow(x-5,y-3) \\ (-1,6)\rightarrow(-1-5,6-3)=(-6,3) \\ (2,4)\rightarrow(2-5,4-3)=(-3,1) \\ (1,2)\rightarrow(1-5,2-3)=(-4,-1) \end{gathered}\)

STEP 4: Plot these new points also

Hence, the type of mapping here is One-to-One

Draw the image located at (-1,6), (2,4), and (1,2). Then for the following mapping. (x,y)>(x-5,y-3).
Draw the image located at (-1,6), (2,4), and (1,2). Then for the following mapping. (x,y)>(x-5,y-3).

The profit function, p(x), for a company is the cost function, c(x), subtracted from the revenue function, r(x). The profit function for the Goto Corporation is p(x) = -0.2x^2 + 250x - 550 and the revenue function is r(x) = -0.1x^2 + 225x. The cost function for the Goto Corporation is

Answers

The cost function for the Goto Corporation is C(x) = 0.1x² - 25x + 550

How to determine the cost function?

From the question, we have the following equations that can be used in our computation:

Profit function, p(x) = -0.2x^2 + 250x - 550

Revenue function, r(x) = -0.1x^2 + 225x

These functions can be expressed as

Profit function, p(x) = -0.2x² + 250x - 550

Revenue function, r(x) = -0.1x² + 225x

The cost function is the difference between the revenue and the profit

So, we have

C(x) = R(x) - P(x)

Substitute the known values in the above equation, so, we have the following representation

C(x) = -0.1x² + 225x + 0.2x² - 250x + 550

Evaluate the like terms

C(x) = 0.1x² - 25x + 550

Hence. cost function is C(x) = 0.1x² - 25x + 550

Read more about cost, revenue and profit functions at

https://brainly.com/question/12983911

#SPJ1

How many ways can you make change for 45 cents using only Nicklesdimes and quarters

Answers

9 nickels,

1 quarter and 4 nickels

only 2

The perimeters of two squares in the model are given. Find the perimeter of the third square. P=12 units p=16 units ​

The perimeters of two squares in the model are given. Find the perimeter of the third square. P=12 units

Answers

Answer:

20 units

Step-by-step explanation:

1) find the side of the first square

side = P / 4 = 12:4 = 3 units

2) find the side of the second square

side = P / 4 = 16 : 4 = 4 units

3) use the Pythagorean theorem to find the hypotenuse of the right triangle

\(\sqrt{4^2 + 3^2} = \sqrt{16 + 9} = \sqrt{25} = 5\) units

4)  find the perimeter of the third square

P = side x 4 = 5 x 4 = 20 units

Answer:

20 units

Step-by-step explanation:

Mark has 11 shirts and 6 pairs of pants. How many different outfits are possible?

Answers

Answer: 66

Step-by-step explanation:

The line L is shown on the grid.
Find an equation for L.
Give your answer in the form y=mx+c

The line L is shown on the grid. Find an equation for L.Give your answer in the form y=mx+c

Answers

Answer:

y = -5x - 5

Step-by-step explanation:

Given the standard form of an equation as y=mx+c

m is the slope

c is the intercept

Using the coordinate points

(1, 0) and (0,-5)

m = -5-0/0+1

m = -5/1

m = -5

Get the intercept

Intercept is the point where thee line cuts the y axis. intercept is -5

Get the equation:

Recall that y = mx+c

y = -5x + -5

y = -5x - 5

This gives the required equation

I need help on this ASAP. It is due in less than 20 minutes so please help! Thanks.

I need help on this ASAP. It is due in less than 20 minutes so please help! Thanks.

Answers

Domain {5,6,7,8,9,10}
Range{3,4,5,6,7,8}

Plzzz helpppppppppppppp

Plzzz helpppppppppppppp

Answers

Answer

Center: (1,-12)

Radius: 5

Work

\(x^{2}\)+\(y^{2}\)-2x+24y+120=0 (Subtract 120 on both sides and rearrange)

\(x^{2}\)-2x+\(y^{2}\)+24y=-120 (Then divide 2 by 2 and square it. Do the same for 24.)

\(x^{2}\)-2x+1+\(y^{2}\)+24y+144=-120+1+144 (You have to add the same numbers on both sides)

\((x-1)^{2} +(y+12)^2=25\) (Simplify)

So you would put (1,-12); r=5

the value of “y” varies directly with “x”. if y= 56, then x= 4

Answers

I'm not sure what the question is here, but they have a simplified ratio of 1:14 (x:y) if it's a direct relationship.

a plumer charges $25 for a service call plus $50 per hour of service. write an equation in slope - intercept form for cost , c ,after h hours of service, what will be the total cost for 8 hours of work? 10 hours of work

Answers

The total cost for 10 hours of work would be 25 + 50(10) = 525.

What is cost?

Cost is the monitorial associated with the purchase of production of food or service if can include the prices of material of which is over it and other expenses that are related to the production of the code of service cost is a major fraction of when deciding whether to purchase a product something as it is and indicator of the value of the product of sound service.
The marginal cost of producing 5 items can be calculated using the equation C(x)=1300+4x/10. The marginal cost is the cost associated with producing one additional item. To calculate the marginal cost, we can plug in x=5 into the equation.

The equation in slope-intercept form for cost, c, after h hours of service is c = 25 + 50h.

The total cost for 8 hours of work would be 25 + 50(8) = 425.

The total cost for 10 hours of work would be 25 + 50(10) = 525.

To know more about cost click-
https://brainly.com/question/25109150
#SPJ1

How to find proportional parts in triangles and parallel lines?

Answers

If a line parallel to one side of a triangle contacts the other two sides of the triangle, the line proportionally splits these two sides. If DE = BC, then ADDB=AEEC.

If a line parallel to one side of a triangle contacts the other two sides, the two sides are proportionately divided.

The formula for a proportional equation is y = kx. The letters y and x denote the variables in the equation. The letter k represents the proportionality constant, which is fixed.

Because matching angles produce the AA similarity shortcut, when a line is drawn parallel to one side of a triangle, two similar triangles are generated. Because the triangles are comparable, the parallel line segments are proportionate segments.

For more questions on Parallel lines

https://brainly.com/question/24607467

#SPJ4

How to find proportional parts in triangles and parallel lines?

John spent 80% of his money and saved the rest. Peter spent 75% of his money and saved the rest. If they saved the same amount of money, what is the ratio of John’s money to Peter’s money? Express your answer in its simplest form.

Answers

The ratio of John's money to Peter's money is 5/4. This means if John has a total amount of 5 then Peter will have a total of 4 as his amount.

Let's assume John has 'x' amount of  money, Peter has 'y' amount of money, The money John saved is 'p' and the money Peter saved is 'q'

So,

p = x - 80x/100                (equation 1)

q = y - 75y/100                (equation 2)

According to the given question, the amount John saved is equal to the amount Peter saved. Hence, we can equate equations 1 and 2.

p = q

x- 80x/100 = y - 75y/100

x - 0.8x = y - 0.75y

0.2x = 0.25y

x =  0.25y/0.2

x/y = 0.25/0.2

x/y = 25/20

x/y = 5/4

Hence, the ratio of John's money to Peter's money is 5/4.

To learn more about Ratio:

https://brainly.com/question/13419413

Solve for :
- 2x + 19 = 3x - 1
-
STEP 1: Add/Subtract one of the X-terms from both sides and re-write the new equation:

Solve for :- 2x + 19 = 3x - 1-STEP 1: Add/Subtract one of the X-terms from both sides and re-write the

Answers

Answer:

x=4

Step-by-step explanation:

-2x+19=3x-1

+2x        +2x

19=5x-1

+1       +1

20=5x

/5   /5

4=x

x=4

A drug is administered intravenously at a constant rate of r mg/hour and is excreted at a rate proportional to the quantity present, with constant of proportionality k>0

.

(Set up and) Solve a differential equation for the quantity, Q
, in milligrams, of the drug in the body at time t hours. Assume there is no drug in the body initially. Your answer will contain r and k.
Q=

Graph Q
against t. What is Q∞, the limiting long-run value of Q?
Q∞=

If r
is doubled (to 2r), by what multiplicative factor is Q∞ increased?
Q∞ (for 2r) = Q∞ (for r

)

Similarly, if r
is doubled (to 2r), by what multiplicative factor is the time it takes to reach half the limiting value, 12Q∞, changed?
t (to 12Q∞), for 2r) = t (to 12Q∞), for r)

If k
is doubled (that is, we use 2k instead of k), by what multiplicative factor is Q∞ increased?
Q∞ (for 2k) = ? Q∞ (for k)

On the time to reach 12Q∞?

t (to 12Q∞), for 2k) = t (to 12Q∞), for k)

Answers

The  differential equation for the quantity,  Q' = -kQ + r; Q = r/k (1-\(e^{-kt}\) ) and the limiting long-run value of Q = x/k

What is  limiting long-run value?

A drug is administered intravenously at a constant rate of r mg/hour and is excreted at a rate proportional to the quantity present, with a constant of proportionality k > 0.

Remember that the differential equation is an equation that contains the derivative of the unknown function.

a) It is given that the rate of change is directly proportional to the quantity dQ/dt ∝ Q

dQ/dt ∝ Q

dQ/dt = -kQ

So, Q' = -kQ + r

where quantity Q is increasing with constant rate r and k is unknown constant.

b) Q' = -kQ + r

dQ/dt = -kQ + r

log(-kQ + r) = -kt -kC

By taking exponential both side, we get

-kQ + r = e power kt+a

-kQ = -r + \(e^{-kt +a}\)

-Q /k= -r/k + /1/kBe\(^{-kt +a/k}\)

Q = -r/k -1/k Be\(^{-kt}\)

At t = 0, Q = 0

1/KB = r/k

B = r

By substituting the value in the above equation

Q = r/k (1-e\(^{-kt}\) )

c) The limiting long-run value of Q, therefore, Q is r/l

Study more about  limiting long-run value on https://brainly.com/question/28659811

#SPJ1

The value Q =

factorize the following using difference between squares

2k³ - 3k²-2k+3

Answers

Answer: The given expression can be factorized as (2k - 3)(k² - 1) using the difference between squares.

Step-by-step explanation: To factorize the given expression using the difference between squares, we need to identify terms that can be written as the square of some other term.

We can rewrite the expression as:

(2k³ - 3k²) - (2k - 3)

Now, we can factor out the common terms from each bracket:

k²(2k - 3) - 1(2k - 3)

We can see that both brackets have a common factor of (2k - 3), which we can factor out:

(2k - 3)(k² - 1)

Therefore, the given expression can be factorized as (2k - 3)(k² - 1) using the difference between squares.

Can somebody help me, please

Can somebody help me, please

Answers

Answer:

uh I don't even know what this is

Answer: x = 500

Step-by-step explanation:

Use the Pythagorean Theorem

a²+b²=c²

300² + 400² = x²

90000 + 160000 = x²

250000 = x²

√250000 = √x²

500 = x

hope i explained it :)

A small software company will bid on a major contract. It anticipates a profit of $10,000 if it gets it, but thinks there is only a 40% chance of that happening. a) What's the expected profit? b) Find the standard deviation for the profit. a) The expected profit is $ (Round to the nearest dollar as needed.) b) The standard deviation is $ (Round to the nearest dollar as needed.)

Answers

a)The expected profit is $4,000. b)The standard deviation for the profit is approximately $4,898.98.

a) To calculate the expected profit, we multiply the profit by the probability of it happening:

Expected profit = Profit * Probability

Given:

Profit = $10,000

Probability = 40% = 0.4

Expected profit = $10,000 * 0.4 = $4,000

Therefore, the expected profit is $4,000.

b) To find the standard deviation for the profit, we need more information about the probability distribution of the profit. If we assume that the profit follows a Bernoulli distribution (with only two possible outcomes: getting the contract or not), we can calculate the standard deviation using the formula:

Standard deviation = √(Probability * (1 - Probability) * Profit^2)

Given:

Profit = $10,000

Probability = 40% = 0.4

Standard deviation = √\((0.4 * (1 - 0.4) * $10,000^2)\)

                = √(0.4 * 0.6 * $100,000,000)

                = √(24,000,000)

                ≈ $4,898.98

Therefore, the standard deviation for the profit is approximately $4,898.98.

Learn more about the standard deviation visit:

brainly.com/question/475676

#SPJ4


- The four-digit number 1210 has an interesting property.
The first digit, 1, counts how many zeros are in the number.
The second digit, 2, counts how many ones are in the number.
The third digit, 1, counts how many twos are in the number.
The fourth digit, 0, counts how many threes are in the number.
Another number with this property is 2020 (2 zeros, 2 twos, no ones or threes.) Now,
make a 7 digit number ABCDEFG so that its first digit, A, counts the number of zeros
in the number, the second digit, B, counts the number of ones, and so on down to G,
which counts the number of sixes in the number

Answers

Answer:

ABCDEFG = 3211000

Step-by-step explanation:

A counts the number of zeroes, and there are 3 zeroes (E, F, G), so A = 3.B counts the number of ones, and there are 2 ones (C, D), so B = 2.C counts the number of twos, and there is 1 two (B), so C = 1.D counts the number of threes, and there is 1 three (A), so D = 1.There is no four, five, six in, so E, F, G are all zeroes.

PLEASE HELP ME I AM PAYING 100 POINTS!!

Mal works at a photo gallery. He charges $50 for a large photo and $40 for a large frame. Sales tax is 5%. How much total tax will a customer pay on both?
Answer the questions to show how to write and simplify expressions that represent the problem.



1. Find the total tax using the expanded expression. Did you get the same answer as you did by evaluating the original expression?


Write your answer in the space below.




2. Which expression, the original expression or the expanded expression, involves finding the total cost first, then calculating the tax on that total?


Write your answer in the space below.


no scams!

Answers

the total tax is 4.50 dollars, yes it is the same because they are the basically the same equation .

the original expression
1. Both the expanded and original have the tax pf $4.50

2. The original
:)

A sequence is defined resursively by the formula f(n + 1) = f(n) + 3. The first term of the sequence is -4. What is the next term in the sequence? -7, -1, 1, 7 ​

Answers

The next term of the sequence is -1.

Given, a sequence is defined recursively by the formula

f(n + 1) = f(n) + 3.

The first term of the sequence is -4.

We, have to find the next term of the sequence,

On using the formula of the sequence, we get

f(n + 1) = f(n) + 3

As, f(1) = -4

Put n = 1,

f(1 + 1) = f(1) + 3

f(2) = f(1) + 3

f(2) = -4 + 3

f(2) = -1

So, the next term of the sequence is -1.

Hence, the next term of the sequence is -1.

Learn more about Sequence and Series here https://brainly.com/question/26748083

#SPJ9

PLEASE HELP MEEE!!! Algebra 2.HELP
Patrick recently bought food for his family reunion. He bought a total of 23 hot dog and hamburger packs. The hot dogs cost $3 each and the hamburgers cost $5 each, he spent a total of $95. How many hot dogs and hamburgers did he buy each?
Write this as a linear equation (_x+_y=_) There should be two of them. PLEASE HELP ME! ITS MY LAST QUESTION ON THISSS

Answers

Answer:

3(x)+5(y) = 95

Step-by-step explanation:

I'm pretty sure this is correct, maybe double check, but I hope this helps :)

X is the amount of hotdogs bought, and y is the amount of hamburgers bought.

how many samples of size n=2 can be drawn from this population

Answers

The samples of size n = 2 that can be drawn from this population is 28

How many samples of size n=2 can be drawn from this population

From the question, we have the following parameters that can be used in our computation:

Population, N = 8

Sample, n = 2

The samples of size n = 2 that can be drawn from this population is calculated as

Sample = N!/(n! * (N - n)!)

substitute the known values in the above equation, so, we have the following representation

Sample = 8!/(2! * 6!)

Evaluate

Sample = 28

Hence, the number of samples is 28


Read more about sample size at

https://brainly.com/question/17203075

#SPJ1

Complete question

A finite population consists of 8 elements.

10,10,10,10,10,12,18,40

How many samples of size n = 2 can be drawn this population?

Which is a correct rotation statement?
a. Rotate triangle ABC 90 degrees clockwise using center A.
b. Rotate triangle ABC 30 degrees counterclockwise using center B.
c. Rotate triangle ABC 180 degrees around center B.
d. Rotate triangle ABC clockwise around center C by 90 degrees
e. Only choices a and b are correct
f. All choices a, b, c, d are correct.
g. There are no correct rotation statements given

Answers

Answer: e

Step-by-step explanation:

because you need the degrees the center and if it is either clockwise or counter clockwise

Answer:

the answer would be 484

step-by-step explanation:

just multiply 0.484 times 1,000 (formula)

Step-by-step explanation:

Other Questions
TRUE/FALSE the transport of carbon dioxide will tend to lower blood ph. the inventors of a disruptive technology typically benefit the most from the technology; it is rare that fast followers catch up quickly. which two organs were described in lecture as the primary sites for gluconeogenesis during fasting (days, not hours, since last meal) What is the main issue in McCulloch v. Maryland? For your Portfolio Project, you will be creating a Training Manual that reflects the focus of each course in the BOA program. Each part of the portfolio will be a chapter in that manual. The purpose of the portfolio is to explain information that organizational administration professionals need to do their jobs well.Portfolio Project Part 4 Description:Ongoing professional development is critical for many reasons, including learning new skills for career growth. It also helps employees keep their skills and knowledge current and enables them to do their jobs better, benefitting the company. This section of the training manual of your portfolio should help let employees know why professional development is important and how one could pursue it. Review Chapter 14 pp. 491-498 of your textbook and respond to the following:In your own words, write about why employees should pursue professional development. Provide at least 3 reasons. Also, include at least ONE brief story as an example of a company employee who took professional development courses and was able to grow his/her career.List your companys professional development requirements. Include the following:Amount required with deadline(s): Decide how to calculate this (i.e., Time? Credits? Another way?)Types of appropriate professional development activities. Formal or informal training? Which formats? List at least THREE.How employees will show they gained knowledge from their professional development activitiesBriefly explain at least THREE ways employees can find professional development opportunities. Consider a variety of methods for developing and improving job skills. (i.e. Traditional college classes, online courses, etc.) If you were to design a website for a team, club, or other group that you belong to, which would you choose: code it yourself with HTML/CSS/JavaScript or use GUI web authoring tools? Why? Find the diameter of the circle. please. Anzio, Inc., has two classes of shares. Class B has 5 times the voting rights as Class A. If you own 12% of the Class A shares and 20% of the Class B shares, what percentage of the total voting rights do you hold? What type of vegetation dominates the basin of the amazon river in south america?. You come in contact with germs/ bacteria internal or external? 36 push ups to 2 minutes unit rate quien tuvo la primera preocupacin por defenir las caractersticas del arte griego? a)homero b)aristotles Find the general form of the quadratic function that has a vertex of (-4,3) and a point on the graph (5,84) The sun is 150,000,000 km from earth; its diameter is 1,400,000 km. A student uses a 5.4-cm-diameter lens with f = 10 cm to cast an image of the sun on a piece of paper.What is the intensity of sunlight in the projected image? Assume that all of the light captured by the lens is focused into the imageThe intensity of the incoming sunlight is 1050 W/m what does the text recommend a company should do before implementing an affirmative action plan Which process allows water to be pulled from the roots to the leaves so it can be used for photosynthesis?. Are the following two expressions equivalent? Explain how you know...3(4-x)6(x+2)-7x You establish a straddle on Fincorp using September call and put options with a strike price of $80. The call premium is $7.00 and the put premium is $8.50. a. What is the most you can lose on this position Is this right? I need help with this question a 30-year-old woman presents with vaginal bleeding and light-headedness. she states that she has soaked four sanitary pads. her bp is 90/50 mm hg, her pulse rate is 120 beats/min, and her respiratory rate is 24 breaths/min. what should you do?